1answer.
Ask question
Login Signup
Ask question
All categories
  • English
  • Mathematics
  • Social Studies
  • Business
  • History
  • Health
  • Geography
  • Biology
  • Physics
  • Chemistry
  • Computers and Technology
  • Arts
  • World Languages
  • Spanish
  • French
  • German
  • Advanced Placement (AP)
  • SAT
  • Medicine
  • Law
  • Engineering
tensa zangetsu [6.8K]
3 years ago
8

Channing jogs 10 miles a week how many miles will she jog in 52 weeks

Mathematics
1 answer:
maria [59]3 years ago
6 0
520 miles. Easy Peasy :)
You might be interested in
DOES ANYONE KNOW THIS???!?!?​
ololo11 [35]

Answer:

15%

Step-by-step explanation:

56-32/32×100

24/32×100=75

100-75=15%

8 0
3 years ago
Read 2 more answers
20. Carmen can buy bottles of paint for $2.00 each and boxes of colored pencils for $3.50 each. She can spend no more than
EleoNora [17]

Answer:

(a) The equality that express many bottles of paint, x, and boxes of colored pencils, y, Carmen can buy is  2 x + 3.5 y = 42

b)Three different solutions:

x  = 14, y = 4 is First Solution.

x  = 7, y = 8 is Second Solution.

x  = 21, y = 0 is Third Solution.

Step-by-step explanation:

Here, the cost of 1 bottle of paint = $2.00 each

The cost of 1 box of colored pencils  = $3.5 each

Let us assume the number of bottle of paints purchased = x

So, the cost of x bottle of paints   = x ( Cost of 1 bottle of paint)

= x ($2.00)  = 2 x

Also, assume the number of box of colored pencils purchased = y

So, the cost of y box of colored pencils = y ( Cost of 1 box)

= y ($3.50)  = 3.5 y

Also, the total amount to be spent on art supplies  = $42

So, the total amount spent on x paint bottles + y box of colored pencil

= $42

or,  2 x + 3.5 y = 42

a ) So, the equality that express many bottles of paint, x, and boxes of colored pencils, y, Carmen can buy is  2 x + 3.5 y = 42

b)Three different solutions:

When y = 4 ,  equation is:   2 x + 3.5(4)  = 42

or,  2 x = 42 - 14  = 28, or x = 28/2  =  14

So, x  = 14, y = 4 is First Solution.

When y = 8 ,  equation is:   2 x + 3.5(8)  = 42

or,  2 x = 42 - 28  = 14, or x = 14/2  =  7

So, x  = 7, y = 8 is Second Solution.

When y = 0 ,  equation is:   2 x + 3.5(0)  = 42

or,  2 x = 42 , or x = 42/2  = 21

So, x  = 21, y = 0 is Third Solution.

7 0
3 years ago
Greatest To Least -5,1, -7, 0, 6
g100num [7]

Answer:

6,1,0,-5,-7

Step-by-step explanation:

8 0
3 years ago
Read 2 more answers
Two angles are supplementary to each other. If the first angle measures 58 then which of the following could be the measure of t
34kurt

Answer: 122°

Step-by-step explanation:

When an angle is supplementary to each other , it means the angle measures to 180°.

Now since one of the two angles is 58°, therefore, the other angle will be

180° - 58°

= 122°

6 0
3 years ago
Evaluate the expression 5x4+2x6
vodka [1.7K]

Answer:

The answer to this is: 132

Step-by-step explanation:

4 0
3 years ago
Read 2 more answers
Other questions:
  • Which system of linear inequalities is graphed?
    12·2 answers
  • How do u solve 1.2= k over 4
    7·2 answers
  • A rectangle measures 13 cm by 5 cm what is its area
    14·1 answer
  • If AB = 3, AD = 5, and DE = 6, what is the length of BC?<br> BC II DE
    13·2 answers
  • Sam's Fruit Cellar earns a $0.40 profit for each apple that it sells and a $0.40 profit for each banana that it sells. Which exp
    10·1 answer
  • Simplify: 12x3 - 3(2x3 + 4x -1) - 5x + 7
    8·2 answers
  • The length of a rectangular frame is 15 inches, and the width of the frame is 8 inches. What is the length of a diagonal of this
    12·1 answer
  • Find the present worth ofRS.2040 due 4 years hence reckoning interest at 5%p.a.<br>​
    15·2 answers
  • Alisa is investing $6,250 to get ready for college expenses. Her bank offers an annual rate of 6.12% per year and she plans to d
    12·1 answer
  • 5 x 4\9 can you tell me the way to get the answer
    11·1 answer
Add answer
Login
Not registered? Fast signup
Signup
Login Signup
Ask question!